Difficulty with washer method when revolving around axis other than y or x axis.

Click For Summary
The discussion revolves around calculating the volume of a solid formed by revolving a region in the second quadrant, bounded by the curve y=-x^3, the x-axis, and the line x=-1, around the line x=-2. The main challenge is expressing the radii for the washer method when the axis of rotation is not the x or y-axis. Participants emphasize the need to convert the curve into terms of y and determine the correct limits for integration. The confusion stems from adjusting the equations for the big and small radii while ensuring proper integration limits. Overall, the thread highlights the complexities of applying the washer method in this scenario.
bah
Messages
2
Reaction score
0

Homework Statement


The region in the second quadrant bounded above by the curve y=-x^3, below by the x-axis, and on the left by the line x=-1, about the line x=-2


Homework Equations



It's basically the big radius squared minus the small radius squared, integrated in terms of y, and multiply that by pi, I think. But I have a hard time coming up the with expression for that when it's revolved around something other than the x or y axis... Please help?

The Attempt at a Solution



Well, the curve in terms of y is x=(-y)^(1/3). I have a hard time adjusting that so it is the right equation for the big radius, and adjusting the x=-1 so it is the small radius...
 
Physics news on Phys.org
Wow that is an interesting washer o.o.

Well if it's about the line x=-2 and you want to change it to the y -axis. What is the y-axis? x=? how can you change x=-2 to x=(y-axis)?

also look at a graph of this function. What are the limits? x=-1, and x=? You also need these for when you change x=(y-axis).
 
Last edited:
wow I'm really confused by this problem. it's been a while since I've done volumes but i figure i could at least get somewhere.

thinking thinking :)
 
Question: A clock's minute hand has length 4 and its hour hand has length 3. What is the distance between the tips at the moment when it is increasing most rapidly?(Putnam Exam Question) Answer: Making assumption that both the hands moves at constant angular velocities, the answer is ## \sqrt{7} .## But don't you think this assumption is somewhat doubtful and wrong?

Similar threads

  • · Replies 4 ·
Replies
4
Views
2K
  • · Replies 8 ·
Replies
8
Views
2K
  • · Replies 3 ·
Replies
3
Views
3K
Replies
6
Views
3K
Replies
1
Views
1K
  • · Replies 1 ·
Replies
1
Views
2K
  • · Replies 5 ·
Replies
5
Views
2K
  • · Replies 1 ·
Replies
1
Views
2K
  • · Replies 16 ·
Replies
16
Views
3K
  • · Replies 2 ·
Replies
2
Views
1K